Download as pdf or txt
Download as pdf or txt
You are on page 1of 37

FIRST SEMESTER CONTINUOUS ASSESSMENT

SUBJECT: INDUSTRIAL ELECTRONICS


NAME: PETER GBOKODAY KOROMA
ID NO: 19350
SUBMITED TO Ing JOSEPH N WILLIAMS
ELECTRICAL AND ELECTRONIC ENGINEERING

𝑸𝑼𝑬𝑺𝑻𝑰𝑶𝑵 𝟏
𝑓 = 100𝐻𝑧, 𝑉𝐿 = 10𝑉, 𝐼𝐿 = 1𝑚𝐴
ϒ = 0.1%
𝐶 =
𝒂)
𝑆𝑜𝐿𝑈𝑇𝐼𝑂𝑁
1
ϒ =
4𝑅𝐿 𝑓𝐶 √3
1
𝐶 =
4𝑅𝐿 𝑓ϒ√3
𝑉𝐿 10
𝐵𝑢𝑡 𝑅𝐿 = = = 10,000Ω
𝐼𝐿 1 × 10−3
1
𝐶 =
0.1
4 × 10000 × 100 × × 3
100 √
𝐶 = 1.443375673 × 10−4
𝐶 = 144.3375673𝜇𝐹
𝒃)
𝑊ℎ𝑒𝑛 ϒ = 0.01%
1
𝐶 =
4𝑅𝐿 𝑓ϒ√3
1
𝐶 =
0.01
4 × 10000 × 100 × × 3
100 √
1.443375673 × 10−3

𝐶 = 1443.375673𝜇𝐹
𝑸𝑼𝑬𝑺𝑻𝑰𝑶𝑵 𝟐.
𝐷𝑎𝑡𝑎
𝑓 = 100𝐻𝑧, 𝑉𝐿 = 10𝑉, 𝐼𝐿 = 1𝑚𝐴,
ϒ = 0.1%
𝐶 =
𝑎)
1
ϒ =
2𝑅𝐿 𝑓𝐶 √3
1
𝐶 =
2𝑅𝐿 𝑓ϒ√3
𝑉𝐿 10
𝐵𝑢𝑡 𝑅𝐿 = = = 10,000Ω
𝐼𝐿 1 × 10−3
1
𝐶 = = 2.886751346 × 10−3
0.1
2 × 10000 × 100 × × 3
100 √
𝐶 = 288. 751346𝜇𝐹
𝒃)
𝑊ℎ𝑒𝑛 ϒ = 0.01%
1
𝐶 =
2𝑅𝐿 𝑓ϒ√3
1
𝐶 = = 2.886751346 × 10−3
0.01
2 × 10000 × 100 × × 3
100 √

𝐶 = 2886.751346𝜇𝐹

𝑸𝑼𝑬𝑺𝑻𝑰𝑶𝑵 𝟑.
𝐷𝑎𝑡𝑎
𝐶1 = 2.5𝑚𝐹
𝐶2 = 25𝑚𝐹
𝑅 = 2.2𝑘Ω
𝑆𝑜𝑙𝑢𝑡𝑖𝑜𝑛
𝐹𝑜𝑟 𝑚𝑎𝑥𝑖𝑚𝑢𝑚 𝐶 = 25𝑚𝐹
1
ϒ =
2𝑅𝐿 𝑓𝐶 √3
𝜔 377
𝑓 = = = 60𝐻𝑧
2𝜋 2𝜋
1
ϒ =
2 × 2.2 × 103 × 60 × 25 × 10−3 × √3
𝛾 = 8.74773135110−5
ϒ = 87.74 × 10−6
𝛾 = 87.74𝜇
𝐹𝑜𝑟 𝑚𝑖𝑛𝑖𝑚𝑢𝑚 𝐶 = 2.5𝑚𝐹
1
ϒ =
2𝑅𝐿 𝑓𝐶 √3
1
ϒ =
2 × 2.2 × 103 × 60 × 2.5 × 10−3 × √3
ϒ = 8.74773135110−4
ϒ = 874.7710−6
𝛾 = 874.77𝜇

𝑸𝑼𝑬𝑺𝑻𝑰𝑶𝑵 𝟒.

𝐷𝑎𝑡𝑎
𝐶1 = 2.5𝑚𝐹
𝐶2 = 25𝑚𝐹
𝑅 = 2.2𝑘Ω
𝐹𝑜𝑟 𝑚𝑎𝑥𝑖𝑚𝑢𝑚 𝐶 = 25𝑀𝑓
1
ϒ =
4𝑅𝐿 𝑓𝐶 √3
𝜔 377
𝑓 = = = 60𝐻𝑧
2𝜋 2𝜋
1
ϒ =
4 × 2.2 × 103 × 60 × 25 × 10−3 × √3
ϒ = 43.74 × 10−6
𝛾 = 43.74𝜇
𝐹𝑜𝑟 𝑚𝑖𝑛𝑖𝑚𝑢𝑚 𝐶 = 2.5𝑀𝑓
1
ϒ =
4𝑅𝐿 𝑓𝐶 √3
1
ϒ =
4 × 2.2 × 103 × 60 × 2.5 × 10−3 × √3
ϒ = 437.39 × 10−6
𝛾 = 437.39𝜇

𝑸𝑼𝑬𝑺𝑻𝑰𝑶𝑵 𝟓
4-Diode Full-Wave Rectifier (Bridge Rectifier)
Configuration: Uses four diodes arranged in a bridge configuration.
Operation: During both halves of the AC cycle, two diodes conduct, providing
full-wave rectification.
Output Voltage: The output voltage is approximately equal to the peak input
voltage minus two diode drops (each conducting path has two diodes in series).
Peak Inverse Voltage (PIV): Each diode must withstand a peak inverse voltage
equal to the peak input voltage.

2-Diode Full-Wave Rectifier (Centre-Tapped Transformer)


Configuration: Uses two diodes and a centre-tapped transformer.
Operation: During the positive half-cycle of the AC input, one diode conducts
and the other does not. During the negative half-cycle, the roles reverse,
allowing full-wave rectification.

Output Voltage: The output voltage is approximately equal to half the peak
input voltage minus one diode drop.
Peak Inverse Voltage (PIV): Each diode must withstand a peak inverse voltage
equal to twice the peak secondary voltage of the transformer (because each
diode only conducts during one half-cycle, the non-conducting diode sees the
full secondary voltage plus the peak voltage from the other half of the winding).
𝑸𝑼𝑬𝑺𝑻𝑰𝑶𝑵 𝟔
Differences between Full-Wave Rectifier (FWR) and Half-Wave Rectifier
(HWR)
Full-Wave Rectifier (FWR)
Configuration: Can be either a 4-diode bridge rectifier or a 2-diode centre-
tapped rectifier.
Output Frequency: The output frequency is double the input AC frequency. For
a 60 Hz AC input, the output ripple frequency is 120 Hz.
Ripple Voltage: The ripple voltage is lower compared to HWR for the same
load and filtering conditions because the rectification occurs during both halves
of the AC cycle.
Maximum Ripple Voltage: The peak-to-peak ripple voltage is reduced because
the voltage never drops to zero between cycles.
Minimum Ripple Voltage: Generally higher compared to HWR because the
rectified voltage does not fall to zero, leading to a higher average DC level and
smaller ripple.

Half-Wave Rectifier (HWR)


Configuration: Uses a single diode.
Output Frequency*: The output frequency is the same as the input AC
frequency. For a 60 Hz AC input, the output ripple frequency is 60 Hz.
Ripple Voltage: The ripple voltage is higher compared to FWR for the same
load and filtering conditions because rectification occurs only during one half
of the AC cycle.
Maximum Ripple Voltage: The peak-to-peak ripple voltage is higher because
the voltage drops to zero between cycles.
Minimum Ripple Voltage: Lower compared to FWR because the rectified
voltage drops to zero, leading to a lower average DC level and larger ripple.

Summary of Differences
1. Diodes Configuration:
- 4-Diode FWR (Bridge): Requires four diodes.
- 2-Diode FWR (Centre-Tapped): Requires two diodes and a centre-tapped
transformer.
𝑸𝑼𝑬𝑺𝑻𝑰𝑶𝑵 𝟕.
𝐹 = 100𝐻𝑧
𝐼𝐿 = 10𝑚𝐴
𝑉𝐿 = 10𝑉
ϒ = 2%
𝑆𝑜𝑙𝑢𝑡𝑖𝑜𝑛
1
𝐶 =
4𝑅𝐿 𝑓ϒ√3
𝑉𝐿 10
𝐵𝑢𝑡 𝑅𝐿 = = = 1000Ω
𝐼𝐿 10 × 10−3
1
𝐶 = = 7.21687836510−5
2
4 × 1000 × 100 × × 3
100 √
𝐶 = 72.2µ𝐹

𝑄𝑈𝐸𝑆𝑇𝐼𝑂𝑁 8
𝑆𝑂𝐿𝑈𝑇𝐼𝑂𝑁
Vs (t) = 170 sin(377t)V, R = 15Ω, 𝑉𝑚 = 170V
170
𝑉𝑜 𝑉𝑚 𝜋
(𝑎)𝐼𝑜 = = = 𝜋 = 3.60𝐴
𝑅 𝑅 15

𝐼𝑜 = 3.60𝐴
𝑉𝑟𝑚𝑠 𝑉𝑚 170
(𝑏) 𝐼𝑟𝑚𝑠 = = = = 5.66𝐴
𝑅 2𝑅 2(15)
𝐼𝑟𝑚𝑠 = 5.66𝐴
(𝑐)𝑃 = 𝐼 2 𝑅 = 5.662 (15) = 480𝑊
𝑃 = 480𝑊

𝑄𝑈𝐸𝑆𝑇𝐼𝑂𝑁 9
𝑇ℎ𝑒 𝑟𝑚𝑠 𝑣𝑎𝑙𝑢𝑒 𝑜𝑓 𝑡ℎ𝑒 𝑠𝑜𝑢𝑟𝑐𝑒 𝑉𝑜𝑙𝑡𝑎𝑔𝑒 𝑖𝑠 240𝑉
𝑇ℎ𝑢𝑠 𝑡ℎ𝑒 𝑟𝑚𝑠 𝑣𝑜𝑙𝑡𝑎𝑔𝑒 𝑎𝑐𝑟𝑜𝑠𝑠 𝑡ℎ𝑒 𝑝𝑟𝑖𝑚𝑎𝑟𝑦 𝑤𝑖𝑛𝑑𝑖𝑛𝑔 𝑖𝑠 𝑉 𝑝𝑟𝑖𝑚𝑎𝑟𝑦
= 240𝑉
𝑉 𝑠𝑒𝑐𝑜𝑛𝑑𝑎𝑟𝑦 𝑖𝑠 𝑡ℎ𝑒 𝑟𝑚𝑠 𝑣𝑜𝑙𝑡𝑎𝑔𝑒 𝑎𝑐𝑟𝑜𝑠𝑠 𝑡ℎ𝑒 𝑠𝑒𝑐𝑜𝑛𝑑𝑎𝑟𝑦 𝑤𝑖𝑛𝑑𝑖𝑛𝑔
𝑉𝑚
𝑉 𝑠𝑒𝑐𝑜𝑛𝑑𝑎𝑟𝑦 =
√2
𝑉𝑜
𝐼𝑜 = 12𝐴, 𝐼𝑜 = ⇒ 𝑉𝑜 = 𝐼𝑜 𝑅 = 12(20) = 240𝑉
𝑅
𝑉𝑚
𝑉𝑜 = , 𝑉 = 𝑉𝑜 𝜋 = 240𝜋 = 754𝑉
𝜋 𝑚
𝑉𝑜 754
𝑉𝑟𝑚𝑠 = = = 533𝑉
√2 √2
𝑇𝑈𝑅𝑁 𝑅𝐴𝑇𝐼𝑂
𝑁1 𝑉𝑜 240
= = = 0.45
𝑁2 𝑉𝑟𝑚𝑠 533

𝑁1 12
(𝑏) 𝐼𝑜 = 𝐼𝑜 = = 26.7𝐴
𝑁2 0.45
𝐼𝑜 = 26.7𝐴

𝑄𝑈𝐸𝑆𝑇𝐼𝑂𝑁 10
𝑉𝑠 = 120 𝑉𝑟𝑚𝑠
𝑓 = 60𝐻𝑧
𝑅 = 12Ω
𝐿 = 12𝑚𝐻
𝑆𝑜𝑙𝑢𝑡𝑖𝑜𝑛
𝑉𝑚 = √2 × 120 = 169.7𝑉 , 𝐿 = 0.012𝐻, 𝜔 = 377𝑟𝑎𝑑𝑠
𝑉𝑚 −𝜔𝑡
(𝑎) 𝑖(𝑡) = [sin(𝜔𝑡 − 𝜙) + sin 𝜙𝑒 𝜔𝜏 ]
𝑍
𝑉𝑠 = 120𝑣𝑟𝑚𝑠 = 𝑉𝑚 = 120𝑣𝑟𝑚𝑠√2 =
𝑓 = 60𝐻𝑧, 𝑅 = 12 𝑜ℎ𝑚, 𝐿 = 12𝑚𝐻
𝜔 = 2𝜋𝑓 = 2𝜋 × 60 = 120𝜋

𝑍 = √𝑅2 + 𝜔𝐿2 = √122 + 377(0.012)2 = 12.8ℎ𝑚


𝜔𝐿 377(0.012)
∅ = tan−1 ( ) = tan−1 ( ) = 0.361 𝑟𝑎𝑑
𝑅 12
𝜔𝐿 377(0.012)
𝜔𝜏 = = = 0.377
𝑅 12
𝛽 = 𝜋 + 𝜃 = 3.50 𝑟𝑎𝑑 = 201°
120√2 −𝜔𝑡
𝑖𝜔𝑡 = [sin(𝜔𝑡 − 0.361) + sin 0.361𝑒 0.377 ]
12.8
−𝜔𝑡
(𝑎)𝑖𝜔𝑡 = 13.2 sin(𝜔𝑡 − 0.361) + 4.67 𝑒 0.377
𝑉𝑚 𝛽
(𝑏) = 𝑉𝐷𝐶 = ∫ sin 𝜔𝑡
2𝜋 0
𝑉𝐷𝐶 𝑉𝑚
𝐼𝐷𝐶 = = (1 − cos 𝛽)
𝑅 2𝜋𝑅
𝑉𝑚
𝑉𝐷𝐶 = (1 − cos 𝛽)
2𝜋𝑅
120√2
𝐼𝐷𝐶 = (1 − cos 201° ) = 4.36𝐴
2𝜋 × 15
𝐼𝐷𝐶 = 4.36𝐴
𝑉𝑟𝑚𝑠 2
(𝑐) 𝑃 = = 𝐼𝑟𝑚𝑠 2 𝑅
𝑅

1 𝑉𝑚2 1
𝐼𝑟𝑚𝑠 = ×√ (𝛽 − sin(2𝛽))
√𝑅2 + 𝜔𝐿2 4𝜋 2

1 1202 𝜋 sin(2 × 201)


𝐼𝑟𝑚𝑠 = ×√ (201 − ) = 6.70𝐴
12.8 2𝜋 180 2

𝑃 = 𝐼𝑟𝑚𝑠 2 𝑅 = 6.702 (12) = 538𝑊


𝑃 = 538𝑊
𝑝 538
(𝑑)𝑝𝑓 = = = 0.67
𝑠 120(6.70)

𝑄𝑈𝐸𝑆𝑇𝐼𝑂𝑁 11

𝑉𝑚 −𝜔𝑡
(𝑎) 𝑖(𝑡) = [sin(𝜔𝑡 − 𝜙) + sin 𝜙𝑒 𝜔𝜏 ]
𝑍
𝑉𝑠 = 240𝑣𝑟𝑚𝑠 = 𝑉𝑚 = 240𝑣𝑟𝑚𝑠√2 =
𝑓 = 60𝐻𝑧, 𝑅 = 15 𝑜ℎ𝑚, 𝐿 = 80𝑚𝐻
𝜔 = 2𝜋𝑓 = 2𝜋 × 60 = 120𝜋

𝑍 = √𝑅2 + 𝜔𝐿2 = √152 + 377(0.08)2 = 33.7𝑜ℎ𝑚


𝜔𝐿 377(0.08)
∅ = tan−1 ( ) = tan−1 ( ) = 1.11 𝑟𝑎𝑑
𝑅 12
𝜔𝐿 377(0.08)
𝜔𝜏 = = = 2.01
𝑅 15
𝛽 = 𝜋 + 𝜃 = 3.50 𝑟𝑎𝑑 = 201°
240√2 −𝜔𝑡
𝑖𝜔𝑡 = [sin(𝜔𝑡 − 1.11) + sin 1.11𝑒 2.01 ]
33.7
−𝜔𝑡
(𝑎)𝑖𝜔𝑡 = 10.1 sin(𝜔𝑡 − 1.11) + 9.02 𝑒 2.01
𝛽 = 𝜋 + 𝜃 = 4.35 𝑟𝑎𝑑 = 250°
𝑉𝐷𝐶 𝑉𝑚
𝑏) 𝐼𝑎𝑣𝑔 = = (1 − cos 𝛽)
𝑅 2𝜋𝑅
240√2
𝐼𝑎𝑣𝑔 = (1 − cos 250) = 4.87𝐴
2𝜋(15)
𝐼𝑎𝑣𝑔 = 4.87𝐴
𝑉𝑟𝑚𝑠 2
𝑐) 𝑃 = = 𝐼𝑟𝑚𝑠 2 𝑅
𝑅

1 𝑉𝑚2 1
𝐼𝑟𝑚𝑠 = ×√ (𝛽 − sin(2𝛽))
√𝑅2 + 𝜔𝐿2 4𝜋 2

1 2402 𝜋 sin(2 × 250)


𝐼𝑟𝑚𝑠 = ×√ (201 − ) = 6.84𝐴
33.7 2𝜋 180 2

𝑃 = 𝐼𝑟𝑚𝑠 2 𝑅 = 6.842 (15) = 701𝑊


𝑃 = 701𝑊
𝑃 701
𝑑) 𝑝𝑓 = = = 0.427
𝑠 240(6.84)

𝑄𝑈𝐸𝑆𝑇𝐼𝑂𝑁 12
𝑉𝑟𝑚𝑠 = 240𝑉
𝑓 = 60𝐻𝑧
𝐿 = 75𝑚𝐻
𝑅 = 10Ω
𝑉𝑑𝑐 = 100𝑉
𝑉𝑚 = √2 × 240 = 339.1𝑉 , 𝐿 = 0.075𝐻, 𝜔 = 377𝑟𝑎𝑑𝑠
𝜔𝐿 377 × 0.075
𝜔𝜏 = = = 2.83𝑟𝑎𝑑𝑠
𝑅 10
Ƶ = √𝑅2 × (𝜔𝐿)2 = √102 × (377 × 0.075)2
Ƶ = 30.0Ω
𝜔𝐿
∅ = 𝑡𝑎𝑛−1 ( )
𝑅
377 × 0.075
∅ = 𝑡𝑎𝑛−1 ( ) = 70.590 , 𝜃 = 1.23𝑟𝑎𝑑
10
𝑉𝑑𝑐 100
𝛼 = 𝑆𝑖𝑛−1 ( ) = 𝑆𝑖𝑛−1 ( ) = 17.16𝑜 = 0.299𝑟𝑎𝑑
𝑉𝑚 339.41
𝑉𝑚 𝑉𝑑𝑐 𝜔𝑡
𝑖(𝜔𝑡) = 𝑠𝑖𝑛(𝜔𝑡 − 𝜃) − + 𝐴℮−𝜔𝜏
Ƶ 𝑅
339.41 100 𝜔𝑡
𝑖(𝜔𝑡) = 𝑠𝑖𝑛(𝜔𝑡 − 1.23) − + 𝐴𝑒 −2.83
30.0 10
𝜔𝑡
𝑖(𝜔𝑡) = 11.317 𝑠𝑖𝑛(𝜔𝑡 – 1.23)– 10 + 𝐴𝑒 −2.83
𝑉𝑚 𝑉𝑑𝑐 𝛼
𝑏𝑢𝑡 𝐴 = [− 𝑠𝑖𝑛(𝛼 − 𝜃) − ]℮𝜔𝜏
Ƶ 𝑅
339.41 100 0.299
𝐴 = [− 𝑠𝑖𝑛(0.299 − 1.23) − ]𝑒 2.83
30.0 10
0.299
𝐴 = [− 11.317 𝑠𝑖𝑛(0.299 − 1.23) − 10]𝑒 2.83
𝐴 = 21.20
𝜔𝑡
𝑖(𝜔𝑡) = 11.317 𝑠𝑖𝑛(𝜔𝑡 – 1.23)– 10 + 21.2𝑒 −2.83
𝛽 = 3.94 𝑟𝑎𝑑 = 226°
𝛽
1
𝐼𝑂 = ∫ 𝑖(𝜔𝑡)𝑑(𝜔𝑡)
2𝜋
𝛼
3.94
1 −
𝜔𝑡
𝐼𝑂 = ∫ 11.317 𝑠𝑖𝑛(𝜔𝑡 − 1.23) − 10 + 21.2𝑒 2.83 𝑑(𝜔𝑡)
2𝜋
0.299

𝐼𝑂 = 3.13𝐴, 𝑃𝑑𝑐 = 𝑉𝑑𝑐 𝐼𝑂 = (100)(3.13) = 313W


𝑏) 𝑡ℎ𝑒 𝑝𝑜𝑤𝑒𝑟 𝑎𝑏𝑠𝑜𝑟𝑏𝑒𝑑 𝑏𝑦 𝑡ℎ𝑒 𝑟𝑒𝑠𝑖𝑠𝑡𝑎𝑛𝑐𝑒
2
𝑃𝑅 = 𝐼𝑟𝑚𝑠 ×𝑅

1 𝛽2
𝐼𝑟𝑚𝑠 = √ ∫ 𝑖 (𝜔𝑡)𝑑(𝜔𝑡)
2𝜋 𝛼

1 3.94 𝜔𝑡
𝐼𝑟𝑚𝑠 = √ ∫ [11.317 𝑠𝑖𝑛(𝜔𝑡 − 1.23) − 10 + 21.20𝑒 −2.83 ]2 𝑑(𝜔𝑡)
2𝜋 0.299

𝐼𝑟𝑚𝑠 = 4.81𝐴
2
𝑃𝑅 = 𝐼𝑟𝑚𝑠 × 𝑅 = 4.812 (10) = 231𝑊
𝑃𝑅 + 𝑃𝑑𝑐
𝑃. 𝑓 =
𝑉𝑟𝑚𝑠 × 𝐼𝑟𝑚𝑠
231.36 + 313 544.361
𝑃. 𝑓 = =
240 × 4.81 1154.4
𝑃. 𝑓 = 0.472

𝑄𝑈𝐸𝑆𝑇𝐼𝑂𝑁 13
𝑉𝑟𝑚𝑠 = 120𝑉
𝑓 = 60𝐻𝑧
𝐿 = 120𝑚𝐻
𝑅 = 12Ω
𝑉𝑑𝑐 = 48𝑉
𝑆𝑜𝑙𝑢𝑡𝑖𝑜𝑛
𝑉𝑚 = √2 × 120 = 169.7𝑉 , 𝐿 = 0.12𝐻, 𝜔 = 377𝑟𝑎𝑑𝑠
Ƶ = √𝑅2 × (𝜔𝐿)2 = √122 × (377 × 0.12)2
Ƶ = 46.8Ω
𝜔𝐿 377 × 0.12
𝜔𝜏 = = = 3.77
𝑅 12

𝜔𝐿
𝜃 = 𝑡𝑎𝑛−1 ( )
𝑅
377 × 0.12
𝜃 = 𝑡𝑎𝑛−1 ( ) → 𝑡𝑎𝑛−1 (3.77) → 75.140 , 𝜃 = 1.31𝑟𝑎𝑑
12
𝑉𝑑𝑐 48
𝛼 = 𝑆𝑖𝑛−1 ( ) = 𝑆𝑖𝑛−1 ( ) = 16.43𝑜 = 0.287𝑟𝑎𝑑
𝑉𝑚 169.7
𝑉𝑚 𝑉𝑑𝑐 𝜔𝑡
𝑖(𝜔𝑡) = 𝑠𝑖𝑛(𝜔𝑡 − 𝜃) − + 𝐴𝑒 −𝜔𝜏
Ƶ 𝑅
169.7 48 −
𝜔𝑡
𝑖(𝜔𝑡) = 𝑠𝑖𝑛(𝜔𝑡 − 1.31) − + 𝐴℮ 3.77
46.8 12
𝑉𝑚 𝑉𝑑𝑐 𝛼
𝑏𝑢𝑡 𝐴 = [− 𝑠𝑖𝑛(𝛼 − 𝜃) − ]℮𝜔𝜏
Ƶ 𝑅
169.7 48 0.29
𝐴 = [− 𝑠𝑖𝑛(0.29 − 1.31) − ]℮3.77
46.8 12
0.29
𝐴 = [− 3.63 𝑠𝑖𝑛(0.29 − 1.31) − 4]℮3.77
𝐴 = 7.66
𝜔𝑡
𝑖(𝜔𝑡) = 3.63 𝑠𝑖𝑛(𝜔𝑡 – 1.31) – 4 + 7.66𝑒 −3.77
𝜔𝑡
(𝛽) = 3.63 𝑠𝑖𝑛(𝛽 – 1.31) – 4 + 7.66𝑒 −3.77
𝛽 = 4.06
𝛽
1
∴ 𝐼𝑂 = ∫ 𝑖(𝜔𝑡)𝑑(𝜔𝑡)
2𝜋
𝛼
4.06
1 𝜔𝑡
𝐼𝑂 = ∫ 3.63 𝑠𝑖𝑛(𝜔𝑡 – 1.31) – 4 + 7.66℮−3.77 𝑑(𝜔𝑡)
2𝜋
0.29

𝐼𝑂 = 1.124𝐴
(𝑎) 𝑡ℎ𝑒 𝑝𝑜𝑤𝑒𝑟 𝑎𝑏𝑠𝑜𝑟𝑏𝑒𝑑 𝑏𝑦 𝑡ℎ𝑒 𝑑𝑐 𝑣𝑜𝑙𝑡𝑎𝑔𝑒 𝑠𝑜𝑢𝑟𝑐𝑒
𝑃𝑑𝑐 = 𝐼𝑂 × 𝑉𝑑𝑐
𝑃𝑑𝑐 = 1.124 × 48
𝑃𝑑𝑐 = 54.0𝑊
(𝑏) 𝑡ℎ𝑒 𝑝𝑜𝑤𝑒𝑟 𝑎𝑏𝑠𝑜𝑟𝑏𝑒𝑑 𝑏𝑦 𝑡ℎ𝑒 𝑟𝑒𝑠𝑖𝑠𝑡𝑎𝑛𝑐𝑒
2
𝑃𝑅 = 𝐼𝑟𝑚𝑠 ×𝑅

1 𝛽2
𝐼𝑟𝑚𝑠 = √ ∫ 𝑖 (𝜔𝑡)𝑑(𝜔𝑡)
2𝜋 𝛼

1 4.06 −
𝜔𝑡
𝐼𝑟𝑚𝑠 = √ ∫ [3.63 𝑠𝑖𝑛(𝜔𝑡 − 1.31) − 4 + 7.66℮ 3.77 ]2 𝑑(𝜔𝑡)
2𝜋 0.29

𝐼𝑟𝑚𝑠 = 1.70𝐴
𝑃𝑅 = 1.72 × 12 = 34.7𝑊
(𝑐) 𝑃𝑜𝑤𝑒𝑟 𝐹𝑎𝑐𝑡𝑜𝑟
𝑃𝑠
𝑃. 𝑓 =
𝑆
𝑏𝑢𝑡 𝑃𝑠 = 𝑃𝑅 + 𝑃𝑑𝑐 , 𝑆 = 𝑉𝑟𝑚𝑠 × 𝐼𝑟𝑚𝑠
𝑃𝑅 + 𝑃𝑑𝑐
𝑃. 𝑓 =
𝑉𝑟𝑚𝑠 × 𝐼𝑟𝑚𝑠
54.0 + 34.7 95.18
𝑃. 𝑓 = =
120 × 1.7 204
𝑃. 𝑓 = 0.435

𝑄𝑈𝐸𝑆𝑇𝐼𝑂𝑁 14
𝑉_𝑟𝑚𝑠 = 120𝑉
𝑓 = 60𝐻𝑧
𝐿 = 100𝑚𝐻 → 0.10𝐻
𝑉𝑑𝑐 = 48𝑉
𝑃𝑜𝑤𝑒𝑟 𝑎𝑏𝑠𝑜𝑟𝑏𝑒𝑑 𝑏𝑦 𝑑𝑐 𝑉𝑜𝑙𝑡𝑎𝑔𝑒 𝑠𝑜𝑢𝑟𝑐𝑒
𝑃𝑑𝑐 = 𝐼𝑂 × 𝑉𝑑𝑐
1 𝛽
𝐼𝑂 = ∫ 𝑖(𝜔𝑡)𝑑(𝜔𝑡)
2𝜋 𝛼
𝑉𝑚 𝑉𝑑𝑐
𝑖𝜔𝑡 = sin(𝑐𝑜𝑠𝛼 − 𝑐𝑜𝑠𝜔𝑡) + (𝛼 − 𝜔𝑡)
𝜔𝐿 𝜔𝐿
𝑉𝑚 = √2 × 120 = 169.7𝑉 , 𝐿 = 0.1𝐻
𝑉𝑑𝑐 48
𝛼 = 𝑆𝑖𝑛−1 ( ) = 𝑆𝑖𝑛−1 ( ) = 16.43𝑜 = 0.29𝑟𝑎𝑑
𝑉𝑚 169.7
169.7 48
𝑖(𝜔𝑡) = ⬚(𝑐𝑜𝑠0.29 − 𝑐𝑜𝑠𝜔𝑡) + (0.29 − 𝜔𝑡)
377×0.1 377×0.1

𝑖(𝜔𝑡) = 4.50(0.96 − 𝑐𝑜𝑠𝜔𝑡) + 1.27(0.29 − 𝜔𝑡)


𝑖(𝜔𝑡) = 4.32 − 4.50𝑐𝑜𝑠𝜔𝑡 + 0.368 − 1.27𝜔𝑡
𝑖(𝜔𝑡) = 4.688 − 4.50𝑐𝑜𝑠(𝜔𝑡) − 1.23𝜔𝑡
𝑙𝑒𝑡 𝜔𝑡 = 𝛽 𝑤ℎ𝑒𝑛 𝑓𝑖𝑛𝑑𝑖𝑛𝑔 𝛽
𝑖(𝛽) = 4.688 − 4.50𝑐𝑜𝑠(𝛽) − 1.27(𝛽)
𝛽 = 4.49 𝑟𝑎𝑑 = 257°
1 𝛽
𝐼𝑂 = ∫ 𝑖(𝜔𝑡)𝑑(𝜔𝑡)
2𝜋 𝛼
1 4.49
𝐼𝑂 = ∫ 4.688 − 4.50cos (ωt) − 127ωt 𝑑(𝜔𝑡)
2𝜋 0.29

𝐼𝑂 = 2.01𝐴
𝑃𝑑𝑐 = 𝐼_𝑂 × 𝑉_𝑑𝑐
𝑃𝑑𝑐 = 48 × 2.01
𝑃𝑑𝑐 = 96.48𝑊
𝑄𝑈𝐸𝑆𝑇𝐼𝑂𝑁15
𝑉𝑚 = 200𝑉
𝑅 = 1𝐾Ω
𝐶 = 1000𝜇𝐹
𝜔 = 377
(𝑎) 𝑅𝑎𝑡𝑖𝑜 𝑜𝑓 𝑅𝐶 𝑡𝑖𝑚𝑒 𝑐𝑜𝑛𝑠𝑡𝑎𝑛𝑡 𝑡𝑜 𝑖𝑛𝑝𝑢𝑡 𝑠𝑖𝑛 𝑤𝑎𝑣𝑒 𝑡𝑖𝑚𝑒 𝑝𝑒𝑟𝑖𝑜𝑑
𝜏 = 𝑅𝐶 = 1𝑘𝑜ℎ𝑚 × 1,000𝜇𝐹 = 𝐼𝑆
𝑇ℎ𝑒 𝑝𝑒𝑟𝑖𝑜𝑑 𝑜𝑓 𝑡ℎ𝑒 𝑖𝑛𝑝𝑢𝑡 𝑠𝑖𝑛𝑒 𝑤𝑎𝑣𝑒 𝑖𝑠 𝑔𝑖𝑣𝑒𝑛 𝑏𝑦
1 1 1
𝑇= = =
𝑓 60𝐻𝑧 60𝑠
𝜏 1𝑠
= = 60
𝑇 1
60𝑠
𝑏) 𝜃 = − tan−1 (𝜔𝑅𝐶) + 𝜋 = − tan−1 (377) + 𝜋 = 1.5573 𝑟𝑎𝑑 = 90.15°
𝑉𝑚 sin 𝜃 = 200 sin(90.15° ) = 199.9993
−(2𝜋+𝛼+𝜃)
sin 𝛼 − sin 𝜃𝑒 𝜔𝑅𝐶 = 0 → 𝛼 = 1.391 𝑟𝑎𝑑 = 79.72°
∆𝑉𝑜 = 𝑉𝑚 (1 − sin 𝛼) = 200(1 − sin 1.391) = 3.21𝑉
𝑉𝑚 200
𝛥𝑉𝑜 ≈ = = 3.33𝑉
𝑓𝑅𝐶 (60)(103 )(10−3 )

𝛥𝑉𝑜 ≈ 3.33𝑉
𝑄𝑈𝐸𝑆𝑇𝐼𝑂𝑁 16
𝜏
𝑎) 𝑅 = 100 𝑜ℎ𝑚: 𝜏 = 𝑅𝐶(100)10−3 = 0.1 𝑠: =6
𝑇
𝜃 = − tan−1 (𝜔𝑅𝐶) + 𝜋 = − tan−1 (37.7) + 𝜋 = 1.5973 𝑟𝑎𝑑 = 91.52°
𝑉𝑚 sin 𝜃 = 200 sin(91.52° ) = 199.93
−(2𝜋+𝛼+𝜃)
sin 𝛼 − sin 𝜃𝑒 𝜔𝑅𝐶 = 0 → 𝛼 = 1.0338 𝑟𝑎𝑑 = 59.23°
∆𝑉𝑜 = 𝑉𝑚 (1 − sin 𝛼) = 200(1 − sin 1.0338) = 28.16𝑉
𝑉𝑚 200
𝛥𝑉𝑜 ≈ = = 33.3𝑉
𝑓𝑅𝐶 (60)(100)(10−3 )

𝛥𝑉𝑜 ≈ 33.3𝑉
𝜏
𝑏) 𝑅 = 10 𝑜ℎ𝑚: 𝜏 = 𝑅𝐶(100)10−3 = 0.01 𝑠: =6
𝑇
𝜃 = − tan−1 (𝜔𝑅𝐶) + 𝜋 = − tan−1 (3.77) + 𝜋 = 1.830 𝑟𝑎𝑑 = 104.9°
𝑉𝑚 sin 𝜃 = 200 sin(104.9° ) = 193.3
−(2𝜋+𝛼+𝜃)
sin 𝛼 − sin 𝜃𝑒 𝜔𝑅𝐶 = 0 → 𝛼 = 1.2883 𝑟𝑎𝑑 = 16.5°
∆𝑉𝑜 = 𝑉𝑚 (1 − sin 𝛼) = 200(1 − sin 1.2883) = 143.2𝑉
𝑉𝑚 200
𝛥𝑉𝑜 ≈ = = 33.3𝑉
𝑓𝑅𝐶 (60)(10)(10−3 )

𝛥𝑉𝑜 ≈ 33.3𝑉

𝑄𝑈𝐸𝑆𝑇𝐼𝑂𝑁 17
𝑅 = 1𝐾Ω = 1000𝑜ℎ𝑚, 𝑉𝑟𝑚𝑠 = 120𝑣 ⟹ 𝑉𝑚 = 120√2, 𝐹 = 60𝐻𝑧,
(𝑎) 𝐶 = 4000𝜇𝐹 𝑡ℎ𝑒 𝑝𝑒𝑎𝑘 − 𝑡𝑜 − 𝑝𝑒𝑎𝑘 𝑟𝑖𝑝𝑝𝑙𝑒 𝑜𝑓 𝑡ℎ𝑒 𝑜𝑢𝑡𝑝𝑢𝑡 𝑣𝑜𝑙𝑡𝑎𝑔𝑒
𝑉𝑚 120√2
𝛥𝑉𝑜 = = = 0.707𝑉
𝑓𝑅𝐶 60 × 1000 × 4000 × 10−6
(𝑏) 𝐶 = 20𝜇𝐹
𝑅𝐶 = 0.02
𝜃 = − tan−1 (𝜔𝑅𝐶) + 𝜋
= − tan−1 (3777)(100)(20)(10−6 ) + 𝜋 = 1.703 𝑟𝑎𝑑 = 97.6°
−(2𝜋+𝛼+𝜃)
sin 𝛼 − sin 𝜃𝑒 𝜔𝑅𝐶

𝛼 = 0.5324 𝑟𝑎𝑑 = 30.5°


𝛥𝑉𝑜 = 𝑉𝑚 − 𝑉𝑚 sin 𝛼 = 120√2 − 120√2 sin 0.5324 = 83.6𝑉

𝑐) 𝑊ℎ𝑒𝑛 𝐶 = 4000µ𝐹
√2 × 120
𝛥𝑉𝑜 =
60 × 2.0
𝛥𝑉𝑜 = 1.414𝑉
𝑐) with 𝐶 = 20𝜇𝐹, 𝑅𝐶 = 0.01
𝜃 = − tan−1 (𝜔𝑅𝐶) + 𝜋
= − tan−1 (3777)(500)(20)(10−6 ) + 𝜋 = 1.83 𝑟𝑎𝑑 = 104.9°
−(2𝜋+𝛼+𝜃)
sin 𝛼 − sin 𝜃𝑒 𝜔𝑅𝐶

𝛼 = 0.2883 𝑟𝑎𝑑 = 16.5°


𝛥𝑉𝑜 = 𝑉𝑚 − 𝑉𝑚 sin 𝛼 = 120√2 − 120√2 sin 0.2883 = 121𝑉
𝑄𝑈𝐸𝑆𝑇𝐼𝑂𝑁 18
𝑇𝑜 𝑑𝑒𝑡𝑒𝑟𝑚𝑖𝑛𝑒 𝑡ℎ𝑒 𝑣𝑎𝑙𝑢𝑒 𝑜𝑓 𝑎 𝑓𝑖𝑙𝑡𝑒𝑟 𝑐𝑎𝑝𝑎𝑐𝑖𝑡𝑜𝑟 𝑡𝑜 𝑘𝑒𝑒𝑝 𝑡ℎ𝑒 𝑝𝑒𝑎𝑘 − 𝑡𝑜
− 𝑝𝑒𝑎𝑘 𝑟𝑖𝑝𝑝𝑙𝑒 𝑎𝑐𝑟𝑜𝑠𝑠 𝑡ℎ𝑒 𝑙𝑜𝑎𝑑 𝑡𝑜 𝑙𝑒𝑠𝑠 𝑡ℎ𝑎𝑛 2 𝑉, 𝑤𝑒 𝑐𝑎𝑛 𝑢𝑠𝑒 𝑡ℎ𝑒 𝑓𝑜𝑙𝑙𝑜𝑤𝑖𝑛𝑔 𝑓𝑜𝑟𝑚𝑢𝑙𝑎:
𝑉𝑝
𝐶=
2𝑓𝑅𝐶 × 𝑟𝑖𝑝𝑝𝑙𝑒 𝑣𝑜𝑙𝑡𝑎𝑔𝑒

𝐸𝑥𝑝𝑙𝑎𝑛𝑎𝑡𝑖𝑜𝑛:
𝑤ℎ𝑒𝑟𝑒:
𝐶 𝑖𝑠 𝑡ℎ𝑒 𝑓𝑖𝑙𝑡𝑒𝑟 𝑐𝑎𝑝𝑎𝑐𝑖𝑡𝑎𝑛𝑐𝑒 𝑖𝑛 𝑓𝑎𝑟𝑎𝑑𝑠
𝑉𝑝 𝑖𝑠 𝑡ℎ𝑒 𝑝𝑒𝑎𝑘 𝑣𝑜𝑙𝑡𝑎𝑔𝑒 𝑜𝑓 𝑡ℎ𝑒 𝑎𝑐 𝑠𝑜𝑢𝑟𝑐𝑒 𝑖𝑛 𝑣𝑜𝑙𝑡𝑠
𝑓 𝑖𝑠 𝑡ℎ𝑒 𝑓𝑟𝑒𝑞𝑢𝑒𝑛𝑐𝑦 𝑜𝑓 𝑡ℎ𝑒 𝑎𝑐 𝑠𝑜𝑢𝑟𝑐𝑒 𝑖𝑛 ℎ𝑒𝑟𝑡𝑧
𝑅 𝑖𝑠 𝑡ℎ𝑒 𝑙𝑜𝑎𝑑 𝑟𝑒𝑠𝑖𝑠𝑡𝑎𝑛𝑐𝑒 𝑖𝑛 𝑜ℎ𝑚𝑠
𝑟𝑖𝑝𝑝𝑙𝑒𝑣𝑜𝑙𝑡𝑎𝑔𝑒 𝑖𝑠 𝑡ℎ𝑒 𝑝𝑒𝑎𝑘 − 𝑡𝑜 − 𝑝𝑒𝑎𝑘 𝑟𝑖𝑝𝑝𝑙𝑒 𝑣𝑜𝑙𝑡𝑎𝑔𝑒 𝑖𝑛 𝑣𝑜𝑙𝑡𝑠
𝑠𝑢𝑏𝑠𝑡𝑖𝑡𝑢𝑡𝑖𝑛𝑔 𝑡ℎ𝑒 𝑔𝑖𝑣𝑒𝑛 𝑣𝑎𝑙𝑢𝑒𝑠 𝑖𝑛𝑡𝑜 𝑡ℎ𝑒 𝑓𝑜𝑟𝑚𝑢𝑙𝑎, 𝑤𝑒 𝑔𝑒𝑡
𝐶 = 169.71𝑒 − 6𝐹
𝑇ℎ𝑒𝑟𝑒𝑓𝑜𝑟𝑒, 𝑡ℎ𝑒 𝑓𝑖𝑙𝑡𝑒𝑟 𝑐𝑎𝑝𝑎𝑐𝑖𝑡𝑜𝑟 𝑚𝑢𝑠𝑡 ℎ𝑎𝑣𝑒 𝑎 𝑣𝑎𝑙𝑢𝑒 𝑜𝑓 𝑎𝑡 𝑙𝑒𝑎𝑠𝑡 169.71 𝑚𝑖𝑐𝑟𝑜𝑓𝑎𝑟𝑎𝑑𝑠.
𝑇𝑜 𝑑𝑒𝑡𝑒𝑟𝑚𝑖𝑛𝑒 𝑡ℎ𝑒 𝑎𝑣𝑒𝑟𝑎𝑔𝑒 𝑎𝑛𝑑 𝑝𝑒𝑎𝑘 𝑣𝑎𝑙𝑢𝑒𝑠 𝑜𝑓 𝑑𝑖𝑜𝑑𝑒 𝑐𝑢𝑟𝑟𝑒𝑛𝑡, 𝑤𝑒 𝑐𝑎𝑛 𝑢𝑠𝑒 𝑡ℎ𝑒
𝑓𝑜𝑙𝑙𝑜𝑤𝑖𝑛𝑔 𝑓𝑜𝑟𝑚𝑢𝑙𝑎𝑠:
𝑉𝑝 120√2
𝐴𝑣𝑒𝑟𝑎𝑔𝑒 𝑑𝑖𝑜𝑑𝑒 𝑐𝑢𝑟𝑟𝑒𝑛𝑡 = =
2𝑅 2 × 750
𝑉𝑝 120√2
𝑃𝑒𝑎𝑘 𝑑𝑖𝑜𝑑𝑒 𝑐𝑢𝑟𝑟𝑒𝑛𝑡 = =
𝑅 750
𝑆𝑢𝑏𝑠𝑡𝑖𝑡𝑢𝑡𝑖𝑛𝑔 𝑡ℎ𝑒 𝑔𝑖𝑣𝑒𝑛 𝑣𝑎𝑙𝑢𝑒𝑠 𝑖𝑛𝑡𝑜 𝑡ℎ𝑒 𝑓𝑜𝑟𝑚𝑢𝑙𝑎𝑠, 𝑤𝑒 𝑔𝑒𝑡:
𝐴𝑣𝑒𝑟𝑎𝑔𝑒 𝑑𝑖𝑜𝑑𝑒 𝑐𝑢𝑟𝑟𝑒𝑛𝑡 = 0.113137𝐴
𝑃𝑒𝑎𝑘 𝑑𝑖𝑜𝑑𝑒 𝑐𝑢𝑟𝑟𝑒𝑛𝑡 = 0.226274𝐴

𝑄𝑈𝐸𝑆𝑇𝐼𝑂𝑁 19
𝑉𝑟𝑚𝑠 = 120, 𝐹 = 60𝐻𝑧, 𝑃𝐿 = 50𝑊, 𝐶 = ?
𝛥𝑉𝑜 = 1.5𝑉
2
𝑉𝑜2 𝑉𝑚2 𝑉𝑚2 (120√2)
𝑃≈ ≈ →𝑅= = = 576 𝑜ℎ𝑚
𝑅 𝑅 𝑃 50
𝑉𝑚 120√2
𝑎) 𝐶 = = = 3,270 𝜇𝐹
𝑓𝑅𝛥𝑉𝑜 60(576)(1.5)
𝛥𝑉𝑜 1.5
𝑏) 𝛼 ≈ sin−1 (1 − ) = sin−1 (1 − ) = 1.438 𝑟𝑎𝑑 = 82.4°
𝑉𝑚 120√2
sin 𝛼
𝐼𝐷,𝑝𝑒𝑎𝑐𝑘 = 𝑉𝑚 (𝜔 Ccos 𝛼 + ) = 28.1𝐴
𝑅
𝑉𝑚 120√2
𝐼𝐷,𝑝𝑒𝑎𝑐𝑘 = = = 0.295 𝐴
𝑅 ⬚

𝑄𝑈𝐸𝑆𝑇𝐼𝑂𝑁 20

𝑉𝑚 120√2
𝑎) 𝑉𝑜 = (1 + cos 𝛼) = (1 + cos 45° )
2𝜋 2𝜋
𝑉𝑜 = 46.1𝑉

2
𝑉𝑟𝑚𝑠 𝑉𝑚 𝛼 sin 2𝛼
𝑏) 𝑃 = ; 𝑉𝑟𝑚𝑠 = √1 − +
𝑅 2 𝜋 2𝜋

120√2 0.785 sin 2(0.785)


= √1 − +
2 𝜋 2𝜋

𝑉𝑟𝑚𝑠 = 80.9𝑉
80.92
𝑃= = 65.5𝑊
100
80.9
𝑐) 𝑆 = 𝑉𝑟𝑚𝑠 𝐼𝑟𝑚𝑠 = 120 = 97.1 𝑉𝐴;
100
𝑃 65.5
𝑝𝑓 = = = 0.674
𝑆 97.1
𝑄𝑈𝐸𝑆𝑇𝐼𝑂𝑁 21
𝐺𝑖𝑣𝑒𝑛 𝑡ℎ𝑎𝑡
𝑉𝑠 = 240 𝑉𝑟𝑚𝑠
𝑓 = 60𝐻𝑧,
𝑅𝐿 = 30ohm
(a) 𝐼𝐷𝐶 = 2.5A
𝑉𝑜 = 𝐼𝑜 𝑅 = (2.5)(30) = 75𝑉
𝑉𝑚 2𝜋𝑉𝑜 2𝜋(75)
𝑉𝑜 = (1 + cos 𝛼) → 𝛼 = cos −1 ( − 1) = cos −1 ( − 1)
2𝜋 𝑉𝑚 240√2
𝛼 = 65.5° 𝑜𝑟 1.143𝑟𝑎𝑑

2
𝑉𝑟𝑚𝑠 𝑉𝑚 𝛼 sin 2𝛼
𝑏) 𝑃 = ; 𝑏𝑢𝑡 𝑉𝑟𝑚𝑠 = √1 − +
𝑅 2 𝜋 2𝜋

240√2 1.143 sin 2(1.143)


= √1 − + = 147.6𝑉
2 𝜋 2𝜋

(147.6)2
𝑃= = 726𝑊
30
147.6
𝑐) 𝑆 = 𝑉𝑟𝑚𝑠 𝐼𝑟𝑚𝑠 = (240) ( ) = 1181𝑉;
30
𝑃 726
𝑝𝑓 = = = 0.615 = 61.5%
𝑆 1181
𝑄𝑈𝐸𝑆𝑇𝐼𝑂𝑁 22
𝑅 = 25 𝑜ℎ𝑚, 𝑓 = 60 𝐻𝑧, 𝐿 = 50 𝑚𝐻, 𝑑𝑒𝑙𝑎𝑦 𝑎𝑛𝑔𝑙𝑒 𝑖𝑠 30, 𝑉𝑜 = 120𝑉𝑟𝑚𝑠

𝑍 = √𝑅2 + 𝜔𝐿2

𝑍 = √(25)2 + (27 × 60 × 0.05)2

= √0625 + 355.306

𝑍 = 31.319𝑜ℎ𝑚

𝐷𝑒𝑡𝑒𝑟𝑚𝑖𝑛𝑒 𝑡ℎ𝑒 𝑙𝑜𝑎𝑑 𝑎𝑛𝑔𝑙𝑒, 𝜃


−1
𝜔𝐿 −1
2𝜋 × 60 × 50 × 10−3
𝜃 = tan ⇒ 𝜃 = tan ( )
𝑅 25

𝜃 = 37.0156°, 𝜃 = 0.646 𝑟𝑎𝑑

𝐷𝑒𝑡𝑒𝑟𝑚𝑖𝑛𝑒 𝑡ℎ𝑒 𝑡𝑖𝑚𝑒 𝑐𝑜𝑛𝑠𝑡𝑎𝑛𝑡.

𝐿 50 × 10−3
𝜏= = = 0.002𝑠
𝑅 25
𝐷𝑒𝑡𝑒𝑟𝑚𝑖𝑛𝑒 𝑡ℎ𝑒 𝑑𝑒𝑙𝑎𝑦 𝑎𝑛𝑔𝑙𝑒 𝑖𝑛 𝑟𝑎𝑑𝑖𝑎𝑛𝑠

𝛼 = 30° = 0.524 𝑟𝑎𝑑


𝑉𝑚 𝛼−𝜔𝑡
(𝑎) 𝑖(𝜔𝑡) = [sin(𝜔𝑡 − 𝜃) − sin(𝛼 − 𝜃)𝑒 𝜔𝜏 ]
𝑍

120√2 0.524−𝜔𝑡
𝑖(𝜔𝑡) = [sin(𝜔𝑡 − 0.646) − sin(0.524 − 0.646)𝑒 377×0.002 ]
31.31
0.524 𝜔𝑡
𝑖(𝜔𝑡) = 5.42 sin(𝜔𝑡 − 0.646) − 5.42 × sin(0.524 − 0.646)𝑒 0.754 ℮0.754
𝜔𝑡
𝑖(𝜔𝑡) = 5.42 sin(𝜔𝑡 − 0.646) − 5.42 sin(−0.122)℮0.695 𝑒 0.754
𝜔𝑡
𝑖(𝜔𝑡) = 5.42 sin(𝜔𝑡 − 0.646) + 5.42 × 0.122 × 2 × 𝑒 0.754

𝜔𝑡
𝑖(𝜔𝑡) = 5.42 sin(𝜔𝑡 − 0.646) + 1.322℮0.754

(𝑏) 𝑤ℎ𝑒𝑟 𝛽 = 𝜋 + 𝜃 = 3.79𝑟𝑎𝑑 = 217.15°

1 𝛽
𝑏) 𝐼𝑜 = ∫ 𝑖(𝜔𝑡)𝑑(𝜔𝑡)
2𝜋 𝛼

1 3.79 𝜔𝑡
= ∫ (5.42 sin(𝜔𝑡 − 0.646) + 1.31𝑒 0.754 ) 𝑑(𝜔𝑡)
2𝜋 0.524

1 3.79
= ∫ (5.42[sin(𝜔𝑡) cos(0.646) − cos(𝜔𝑡) sin(0.646)]
2𝜋 0.524
𝜔𝑡
+ 1.31𝑒 0.754 ) 𝑑(𝜔𝑡)
1 3.79 𝜔𝑡
= ∫ (5.42 [0.8 sin(𝜔𝑡) − 0.6 cos(𝜔𝑡)] + 1.31𝑒 0.754 ) 𝑑(𝜔𝑡)
2𝜋 0.524

1 𝜔𝑡 3.79
𝐼𝑜 = [−4.336 cos(𝜔𝑡) − 3.252 sin(𝜔𝑡) + 1.31 × (−0.754) 𝑒 0.754 ]
2𝜋 0.524

1 𝜔𝑡 3.79
= − [4.336 cos(𝜔𝑡) + 3.252 sin(𝜔𝑡) + 0.99𝑒 0.754 ]
2𝜋 0.524

1 3.79
= − [4.336 cos(3.79) + 3.252 sin(3.79) + 0.99𝑒 0.754
2𝜋
0.524
− [4.336 cos(0.524) + 3.252 sin(0.524) + 0.99𝑒 0.754 ]]

1
=− [−5.413 − 5.875]
2𝜋
𝐼𝑜 ≅ 1.8𝐴

1 𝛽2
𝑐) 𝐼𝑟𝑚𝑠 = √ ∫ 𝑖 (𝜔𝑡)𝑑(𝜔𝑡)
2𝜋 𝛼

1 3.79 𝜔𝑡 2
=√ ∫ (5.42 sin(𝜔𝑡 − 0.646) + 1.31℮0.754 ) 𝑑(𝜔𝑡)
2𝜋 0.524

3.79
1 2
2𝜔𝑡 𝜔𝑡
= √ ∫ (29.38 sin (𝜔𝑡 − 0.646) + 1.716𝑒 0.754 + 14.2 sin(𝜔𝑡 − 0.646)𝑒 0.754 ) 𝑑(𝜔𝑡)
2𝜋 0.524

𝐼𝑟𝑚𝑠 = 2.80𝐴
2
𝑃 = 𝐼𝑟𝑚𝑠 𝑅 = 2.802 × 25 = 196𝑊

𝑃 = 196𝑊

𝑄𝑈𝐸𝑆𝑇𝐼𝑂𝑁 23

𝑉𝑠 = 120𝑉𝑟𝑚𝑠, 𝑓 = 60𝐻𝑧, 𝑅 = 40𝑜ℎ𝑚, 𝐿 = 75𝑚𝐻, 𝛼 = 60° , 1.047𝑟𝑎𝑑


−1
𝜔𝐿 −1
377 × 75 × 10−3
𝜃 = tan ( ) = tan ( ) = 35.25°
𝑅 40
𝜃 = 35.25°
𝑉𝑜 𝑉𝑚
(𝑎) 𝐼𝑂 = 𝐵𝑈𝑇 𝑉𝑜 = (cos 𝛼 − cos 𝛽)
𝑅 2𝜋
𝑉𝑚
∴ 𝐼𝑂 = (cos 𝛼 − cos 𝛽)
2𝜋𝑅
𝑉𝑚
(𝑏) 𝐼𝑂 = (cos 𝛼 − cos 𝛽)
2𝜋𝑅
120√2
= (cos 60° − cos 35.25° ) = 0.889𝐴
2𝜋 × 40
𝐼𝑂 = 0.889𝐴

𝑉𝑟𝑚𝑠 = 63.34𝑉
63.34
𝐼𝑟𝑚𝑠 = = 1.50𝐴
40
2
𝑃𝑅 = 𝐼𝑟𝑚𝑠 𝑅 = 1.502 × 40 = 99.85𝑊
𝑃𝑅 = 90.3𝑊

𝑄𝑈𝑆𝑇𝐼𝑂𝑁 24
𝐺𝑖𝑣𝑒𝑛 𝑡ℎ𝑎𝑡
𝑉𝑠 = 120 𝑉𝑟𝑚𝑠, 𝐿 = 40𝑚𝐻
𝑓 = 60𝐻𝑧,
𝑅𝐿 = 20ohm
(a) 𝐼𝐷𝐶 = 2.0A

𝑉𝐷𝐶 = 𝐼𝐷𝐶 × R
𝑉𝑚
𝑅𝐿 = (1 + cos ∝)
2𝜋
120√2
2.0 × 20 = (1 + cos 𝛼)
2𝜋

2.0 × 20 × 2 × 3.14
1 + cos 𝛼 =
√2 × 120

1 + cos 𝛼 =
cos 𝛼 =
𝛼 ≈ 46°

𝑄𝑈𝐸𝑆𝑇𝐼𝑂𝑁 25
𝑅 = 16 Ω, 𝑉𝑠 = 120 𝑉 𝑟𝑚𝑠, 𝐿 = 60 𝑚𝐻,
𝑓 = 60 𝐻𝑧, 𝑎𝑣𝑒𝑟𝑎𝑔𝑒 𝑐𝑢𝑟𝑟𝑒𝑛𝑡 𝑜𝑓 1.8𝐴
𝑉𝑎𝑣𝑔
𝐼𝑎𝑣𝑔 = , 𝑉𝑎𝑣𝑔 = 𝐼𝑎𝑣𝑔 × 𝑅 = 2.0 × 20 = 40𝑉
𝑅
𝑉𝑎𝑣𝑔 = 40𝑉
𝑓𝑜𝑟 𝑡ℎ𝑒 ℎ𝑎𝑙𝑓 − 𝑤𝑎𝑣𝑒 𝑟𝑒𝑐𝑡𝑖𝑓𝑖𝑒𝑟 𝑤𝑖𝑡ℎ 𝑅𝐿 𝑙𝑜𝑎𝑑
𝑉𝑚
𝑉𝑎𝑣𝑔 = (1 + cos 𝛼)
2𝜋
120√2
1.8 = (1 + cos 𝛼) ⇒ 1.8 = 1.688(1 + cos 𝛼)
2𝜋 × 16
1.8
= 1 + cos 𝛼 ⇒ cos 𝛼 ≈ 0.112
1.688
𝛼 = cos −1 (0.112) ≈ 60.5°
𝑡ℎ𝑒 𝑑𝑒𝑙𝑎𝑦 𝑎𝑛𝑔𝑙𝑒 (𝛼)𝑖𝑠 60.5°

𝑸𝑼𝑬𝑺𝑻𝑰𝑶𝑵 𝟐𝟔
𝑉𝑠 = 120 𝑉 𝑟𝑚𝑠, 𝐿 = 100 𝑚𝐻,
𝑅 = 12 Ω, 𝑎𝑛𝑑 𝑉𝑑𝑐 = 48 𝑉, 𝑑𝑒𝑙𝑎𝑦 𝑎𝑛𝑔𝑙𝑒 𝑖𝑠 50
𝑉𝑚 𝑉𝑑𝑐 −𝜔𝑡
𝑖(𝜔𝑡) = sin(𝜔𝑡 − 𝜃) − + 𝐴𝑒 𝜔𝜏
𝑍 𝑅
𝑍 = √𝑅2 + 2𝜋𝑓𝐿2 = √122 + 2𝜋 × 60 × 0.12
𝑍 = 39.5629𝑜ℎ𝑚, 𝜔 = 377𝑟𝑎𝑑
𝜔𝐿
𝜃 = tan−1
𝑅
377 × 0.1 377 × 0.1
𝜃 = tan−1 = 72.34° , 1.262𝑟𝑎𝑑, 𝜔𝜏 = = 3.142
12 12
∝= 50∘ , 0.87𝑟𝑎𝑑
(a) 𝑜𝑛 𝑝𝑢𝑡𝑡𝑖𝑛𝑔 𝑣𝑎𝑙𝑢𝑎𝑠, 𝑤𝑒 𝑔𝑒𝑡
−𝝎𝒕
𝒊(𝝎𝒕) = 4.29 sin(𝜔𝑡 − 1.263) − 4.0 + 7.43𝒆3.142 𝑨
𝑤ℎ𝑒𝑟𝑒 𝐼𝑜 = 1.04𝐴
𝑎𝑛𝑑 𝑃𝑑𝑐 = 𝐼𝑜 × 𝑉𝑑𝑐 = 1.04 × 48
∴ 𝐶 = 50.1𝑊

2
1 𝛽2
(𝑏) 𝑃𝑟 = 𝐼𝑟𝑚𝑠 𝑅, 𝑤ℎ𝑒𝑟𝑒 𝐼𝑟𝑚𝑠 = ∫ 𝑖 (𝜔𝑡)𝑑(𝜔𝑡)
2𝜋 ∝
𝑜𝑛 𝑝𝑢𝑡𝑖𝑛𝑔 𝑡ℎ𝑒 𝑣𝑎𝑙𝑢𝑠, 𝑤𝑒 𝑔𝑒𝑡. 𝐼𝑟𝑚𝑠 = 1.67𝐴
𝑃𝑟 = 1.672 × 12 = 33.5𝑊
𝑃𝑑𝑐 + 𝑃𝑟 50.1 + 33.5
(𝑐)𝑝𝑓 = = = 0.417
𝑉 × 𝐼𝑟𝑚𝑠 120 × 1.67
𝑝𝑓 = 0.417

𝑸𝑼𝑬𝑺𝑻𝑰𝑶𝑵 𝟐𝟕
𝑉𝑠 = 240 𝑉 𝑟𝑚𝑠, 𝐿 = 150 𝑚𝐻,
𝑅 = 100 Ω, 𝑎𝑛𝑑 𝑉𝑑𝑐 = 96 𝑉, 𝑑𝑒𝑙𝑎𝑦 𝑎𝑛𝑔𝑙𝑒 𝑖𝑠 60
𝑉𝑚 𝑉𝑑𝑐 −𝜔𝑡
𝑖(𝜔𝑡) = sin(𝜔𝑡 − 𝜃) − + 𝐴𝑒 𝜔𝜏
𝑍 𝑅
𝑍 = √𝑅2 + 2𝜋𝑓𝐿2 = √1002 + 2𝜋 × 60 × 0.152
𝑍 = 114.88𝑜ℎ𝑚, 𝜔 = 377𝑟𝑎𝑑
𝜔𝐿
𝜃 = tan−1
𝑅
377 × 0.15 377 × 0.15
𝜃 = tan−1 = 29.48° , 1.262𝑟𝑎𝑑, 𝜔𝜏 = = 0.5655
100 100
∝= 60∘ , 𝑟𝑎𝑑
(a) 𝑜𝑛 𝑝𝑢𝑡𝑡𝑖𝑛𝑔 𝑣𝑎𝑙𝑢𝑎𝑠, 𝑤𝑒 𝑔𝑒𝑡
−𝝎𝒕
𝒊(𝝎𝒕) = 2.95 sin(𝜔𝑡 − 0.515) − 0.96 + 3.44𝒆0.565 𝑨
𝛽
1
𝐼𝑂 = ∫ 𝑖(𝜔𝑡)𝑑(𝜔𝑡)
2𝜋
𝛼

𝑤ℎ𝑒𝑟𝑒 𝐼𝑜 = 0.454𝐴
𝑎𝑛𝑑 𝑃𝑑𝑐 = 𝐼𝑜 × 𝑉𝑑𝑐 = 0.454 × 96
∴ 𝑃𝑑𝑐 = 43.6𝑊

2
1 𝛽2
(𝑏) 𝑃𝑟 = 𝐼𝑟𝑚𝑠 𝑅, 𝑤ℎ𝑒𝑟𝑒 𝐼𝑟𝑚𝑠 = √ ∫ 𝑖 (𝜔𝑡)𝑑(𝜔𝑡)
2𝜋 ∝

𝑜𝑛 𝑝𝑢𝑡𝑖𝑛𝑔 𝑡ℎ𝑒 𝑣𝑎𝑙𝑢𝑠, 𝑤𝑒 𝑔𝑒𝑡. 𝐼𝑟𝑚𝑠 = 0.830𝐴


𝑃𝑟 = 0.8302 × 100 = 69.0𝑊
𝑃𝑑𝑐 + 𝑃𝑟 43.6 + 69.0
(𝑐)𝑝𝑓 = = = 0.565
𝑉 × 𝐼𝑟𝑚𝑠 240 × 0.830
𝑝𝑓 = 0.565

𝑸𝑼𝑬𝑺𝑻𝑰𝑶𝑵 𝟐𝟖
𝑉𝑆 = 120𝑉𝑟𝑚𝑠
𝑓 = 60𝐻𝑧
𝐿 = 75𝑚𝐻 → 0.075𝐻
𝑅 = 2Ω
𝑉𝑑𝑐 = 48𝑉
𝛼 = 500
𝑆𝑜𝑙𝑢𝑡𝑖𝑜𝑛
𝑉𝑚 = √2 × 120 = 169.7𝑉 , 𝐿 = 0.075𝐻, 𝜔 = 377𝑟𝑎𝑑𝑠
Ƶ = √𝑅2 × (𝜔𝐿)2 = √22 × (377 × 0.075)2
Ƶ = 28.3Ω
𝜔𝐿 377 × 0.075
𝜔𝜏 = = = 14.14𝑟𝑎𝑑𝑠
𝑅 2
𝜔𝐿
𝛩 = 𝑡𝑎𝑛−1 ( )
𝑅
377 × 0.075
= 𝑡𝑎𝑛−1 ( ) → 𝑡𝑎𝑛−1 (14.14) → , 𝜃 = 1.5𝑟𝑎𝑑
2
𝛼 = 500 = 0.87𝑟𝑎𝑑
(𝑎) 𝑎𝑛 𝑒𝑥𝑝𝑟𝑒𝑠𝑠𝑖𝑜𝑛 𝑓𝑜𝑟 𝑙𝑜𝑎𝑑 𝑐𝑢𝑟𝑟𝑒𝑛𝑡
𝑇ℎ𝑒 𝑚𝑖𝑛𝑖𝑚𝑢𝑚 𝑑𝑒𝑙𝑎𝑦 𝑎𝑛𝑔𝑙𝑒 𝑖𝑠 360 . 𝑤ℎ𝑖𝑐ℎ 𝑖𝑛𝑑𝑖𝑐𝑎𝑡𝑒 𝑡ℎ𝑎𝑡 500 𝑖𝑠 𝑎𝑙𝑙𝑜𝑤𝑎𝑏𝑙𝑒.
𝑉𝑚 𝑉𝑑𝑐 𝜔𝑡
𝑖(𝜔𝑡) = 𝑠𝑖𝑛(𝜔𝑡 − 𝜃) − + 𝐴𝑒 −𝜔𝜏
Ƶ 𝑅
169.7 48 −
𝜔𝑡
𝑖(𝜔𝑡) = 𝑠𝑖𝑛(𝜔𝑡 − 1.5) − + 𝐴𝑒 14.14
28.3 2
𝑉𝑚 𝑉𝑑𝑐 𝛼
𝑏𝑢𝑡 𝐴 = [− 𝑠𝑖𝑛(𝛼 − 𝜃) − ]℮𝜔𝜏
Ƶ 𝑅
169.7 48 0.87
𝐴 = [− 𝑠𝑖𝑛(0.87 − 1.5) − ]𝑒 14.14
28.3 2
𝐴 = 29.1
𝜔𝑡
∴ 𝑖(𝜔𝑡) = 5.99 𝑠𝑖𝑛(𝜔𝑡 – 1.5) – 24 + 29.3𝑒 −14.14
𝑊ℎ𝑒𝑟𝑒 𝑡ℎ𝑒 𝑒𝑥𝑡𝑖𝑛𝑐𝑡𝑖𝑜𝑛 𝑎𝑛𝑔𝑙𝑒 𝛽 𝑖𝑠 𝑓𝑜𝑢𝑛𝑑 𝑛𝑢𝑚𝑒𝑟𝑖𝑐𝑎𝑙𝑙𝑦
𝛽
𝑖(𝛽) = 5.99𝑠𝑖𝑛(𝛽 – 1.5) – 24 + 30.1𝑒 −14.14
𝛽 = 4.226 = 4.23𝑟𝑎𝑑, 242.92
(𝑏) 𝑝𝑜𝑤𝑒𝑟 𝑎𝑏𝑠𝑜𝑟𝑏𝑒𝑑 𝑏𝑦 𝑡ℎ𝑒 𝐷𝑐 𝑣𝑜𝑙𝑡𝑎𝑔𝑒 𝑠𝑜𝑢𝑟𝑐𝑒
𝑃𝑑𝑐 = 𝐼𝑂 × 𝑉𝑑𝑐
𝛽
1
𝐵𝑢𝑡 𝐼𝑂 = ∫ 𝑖(𝜔𝑡)𝑑(𝜔𝑡)
2𝜋
𝛼
4.23
1 𝜔𝑡
𝐼𝑂 = ∫ (5.99 sin(𝜔𝑡 – 1.5) – 24 + 29.1𝑒 −14.14 ) 𝑑(𝜔𝑡)
2𝜋
0.87
1
= (14.4556)
2𝜋
𝐼𝑂 = 1.91𝐴
𝑃𝑑𝑐 = 𝐼𝑂 × 𝑉𝑑𝑐
𝑃𝑑𝑐 = 1.91 × 48
𝑃𝑑𝑐 = 91.6𝑊
(𝑐) 𝑡ℎ𝑒 𝑝𝑜𝑤𝑒𝑟 𝑎𝑏𝑠𝑜𝑟𝑏𝑒𝑑 𝑏𝑦 𝑡ℎ𝑒 𝑟𝑒𝑠𝑖𝑠𝑡𝑎𝑛𝑐𝑒
2
𝑃𝑅 = 𝐼𝑟𝑚𝑠 ×𝑅

1 𝛽2
𝑏𝑢𝑡 𝐼𝑟𝑚𝑠 = √ ∫ 𝑖 (𝜔𝑡)𝑑(𝜔𝑡)
2𝜋 𝛼

1 4.23 −
𝜔𝑡 2
𝐼𝑟𝑚𝑠 = √ ∫ [5.99 𝑠𝑖𝑛(𝜔𝑡 − 1.5) − 24 + 29.1𝑒 14.14 ] 𝑑(𝜔𝑡)
2𝜋 0.87

1
=√ (72.2077)
2𝜋

𝐼𝑟𝑚𝑠 = 2.93𝐴
𝑃𝑅 = (2.93)2 × 2 = 17.1𝑊
𝑃𝑅 = 17.1𝑊

𝑸𝑼𝑬𝑺𝑻𝑰𝑶𝑵 𝟐𝟗
𝑉𝑚 = 100 𝑉, 𝐿 = 35 𝑚𝐻, 𝑉𝑑𝑐 = 24 𝑉, Ꞷ = 2𝜋60 𝑟𝑎𝑑/𝑠, 𝑎𝑛𝑑 𝛼 = 75.
𝑑𝑖(𝑡)
𝐿 = 𝑉𝑚 sin 𝜔𝑡 − 𝑉𝑑𝑐
𝑑𝑡
𝑑𝑖(𝑡) 1
= [𝑉𝑚 sin 𝜔𝑡 − 𝑉𝑑𝑐 ]𝑂𝑅
𝑑𝑡 𝐿
𝑑𝑖(𝜔𝑡) 1
= [𝑉 sin 𝜔𝑡 − 𝑉𝑑𝑐 ]
𝑑(𝜔𝑡) 𝜔𝐿 𝑚
1 𝜔𝑡
𝑖(𝜔𝑡) = ∫ (𝑉𝑚 sin 𝜔𝑡 − 𝑉𝑑𝑐 )𝑑 (𝜔𝑡)
𝜔𝐿 𝛼
𝑉𝑚 𝑉𝑑𝑐
𝑖(𝜔𝑡) = (cos 𝛼 − cos 𝜔𝑡) + (𝛼 − 𝜔𝑡)
𝜔𝐿 𝜔𝐿
100 24
𝑖(𝜔𝑡) = (cos 75 − cos 𝜔𝑡) + (75 − 𝜔𝑡)
377 × 0.035 377 × 0.035
𝑖(𝜔𝑡) = 4.34 − 7.58 cos 𝜔𝑡 − 1.82𝜔𝑡 𝐴, 1.309 ≤ 𝜔𝑡 ≤ 4.249
1 𝛽
𝐼𝑜 = ∫ 𝑖(𝜔𝑡)𝑑(𝜔𝑡)
2𝜋 𝛼
1 4.2
𝐼𝑜 = ∫ 4.34 − 7.58 cos 𝜔𝑡 − 1.82𝜔𝑡 = 1.91 𝐴
2𝜋 1.3
𝐼𝑜 = 1.91 𝐴

𝑸𝑼𝑬𝑺𝑻𝑰𝑵 𝟑𝟎

Data
R1 160Ω

R2

75Ω

V1 R3
120Vrms
60Hz Battery E

V2
48V
Rectifier
Circuit

R160 =which is used when source supplies 160W


R160 = which is used when source supplies 75W

for delivery Dc load average value of rectified voltage average is taken.

Solution

2Vm
Vavg =
π

Vm = peak value, at Source voltage

2 × 120√2
Vavg = = I = 108V
π

Consider when 160W of power is being delivered

48I = 160

160
I=
48

I = 3.33

Vavg − E
From KVL ⟶ I =
R160

108 − 48
I=
R160

108 − 48
R160 =
I
108 − 48
R160 =
3.33

R160 = 18Ω

So, for 75W

48 × I = 75
75
I=
48

I = 1.5625

Vavg − E
From KVL I=
R 75

108 − 48
So, I =
R 75

108 − 48
R160 = 1,5625

R160 = 38.4

Control switches from one resistor to another supply the required power.

Question 31

Given that

Current = 2𝐴 = 𝐼𝑎𝑣𝑔

Inductance = 100mH = L

Ac Source = 120Vrms

F = 60Hz

Solution

We know that the equation

𝛾𝐿 = 𝑗𝜔𝐿 [∵ 𝜔 = 2𝜋𝑓]

𝛾𝐿 = 𝑗2𝜋𝑓𝐿

There,
𝑓 = 60𝐻𝑧

𝐿 − 100 × 10−3

𝛾𝐿 = 𝑗(2𝜋 × 60 × 100 × 10−3 )

𝛾𝐿 = 𝑗(120 × 3.142 × 100 × 10−3

𝛾𝐿 = 𝑗(37.704)Ω

𝛾𝐿 = 𝑗37.704Ω

We know that
𝑉𝑚
𝐼𝑟𝑚𝑠 =
2

Ζ = √𝑅 Υ + (Υ𝑙 − Υ𝑐 )2

Assume that the circuit for given data


2A
L1
I(t)
100mH

Vt

R1

∵ 𝐼𝑟𝑚𝑠 =1.1× 𝐼𝑎𝑣𝑔

But 𝐼𝑎𝑣𝑔 = 2𝐴

𝐼𝑟𝑚𝑠 =1.1× 2

𝐼𝑟𝑚𝑠 = 2.2𝐴
𝑉𝑟𝑚𝑠
𝐼𝑟𝑚𝑠 =
𝑍

120
2.2 =
𝛾
√𝑅Υ + 𝑋𝑙

𝛾 120
√𝑅Υ + 𝑋𝑙 =
2.2
𝛾 120 𝛾
𝑅Υ + 𝑋𝑙 = ( )
2.2

𝑅Υ = (54.55)𝛾 − (𝑗37.704)𝛾

There,

𝛾𝐿 = 𝑗37.704Ω

𝑅Υ = 2975.7 + 1421.591 = 4397.29

𝑅 = √4397.29 = 66.312Ω

𝑅 = 66.312Ω

Alternative circuit to satisfy the design specification

2A

L2
V1 100mH
120Vrms
60Hz

R
66.312Ω
QUESTIN 32.

Solution

D1 D2

N1 : N2
A
Vs RL
B
Co1 Vo

D3 D4

𝑉𝑠 = 120𝑉𝑟𝑚𝑠

𝑉𝑠 = 120 × √2 = 169.70𝑉

𝑉𝑜𝑙𝑡𝑎𝑔𝑒 𝑟𝑒𝑞𝑢𝑖𝑟𝑒𝑑 𝑎𝑡 𝑜𝑢𝑡𝑝𝑢𝑡 = 48𝑉


48 1
𝑇𝑢𝑟𝑛 𝑟𝑎𝑡𝑖𝑜 = =
169.70 3.53

𝐶𝑎𝑝𝑎𝑐𝑖𝑡𝑜𝑟 𝐶𝑜 𝑖𝑠 𝑟𝑒𝑞𝑢𝑖𝑟𝑒𝑑 𝑡𝑜 𝑚𝑎𝑘𝑒 𝑣𝑜𝑙𝑡𝑎𝑔𝑒 𝑉𝑜 𝑠𝑚𝑜𝑜𝑡ℎ

𝐺𝑖𝑣𝑖𝑛𝑔 𝑎 𝑠𝑝𝑒𝑐𝑖𝑓𝑖𝑐𝑎𝑡𝑖𝑜𝑛 𝑜𝑓 1.1 𝑟𝑖𝑝𝑝𝑙𝑒

= 0.01 × 48 = 0.48𝑉 𝑟𝑖𝑝𝑝𝑙𝑒 𝑜𝑛 48𝑉 𝐷𝐶

𝑉𝑜2 𝑉𝑜2
𝐹𝑜𝑟 100𝑊 , = 100 ⟹ 𝑅𝑙 =
𝑅𝑙 100

482
𝑅𝑙 =
100

𝑅𝑙 = 23.04𝛺

Assuming load to be of constant source

Vo

Co

Vo

𝑁2 |𝑉𝑠|
𝑁1

to 𝑡1 𝑇𝑠 𝑡2 𝑡3 𝑇𝑠
2
∆𝑉
𝐶𝑜 = 𝐼𝑜
∆𝑡

𝑡1 ≤ 𝑡 ≤ 𝑡2
3𝑇𝑠 𝑇𝑠
but 𝑡2 = 4
, 𝑡1 =
4

𝑇𝑠
𝐼𝑜 ∆𝑡 𝐼𝑜 × 4
∆𝑉 = =
𝐶𝑜 𝐶𝑜

𝑇𝑠
𝐼𝑜 ×
𝐶𝑜 = 4
∆𝑉
𝑉𝑜 × 𝑇𝑠
𝐶𝑜 =
𝑅𝑙 × 4 × 𝑓𝑠 × ∆𝑉

𝑉𝑜
𝐶𝑜 =
𝑅𝑙 × 4 × 𝑓𝑠 × ∆𝑉

𝑉𝑜
𝐶𝑜 =
23.04 × 4 × 60 × 0.48

𝐶𝑜 = 0.0180𝑓

𝐶𝑜 = 18𝑚𝐹

Alternative Circuits

Vs
Phase Shifted Full Bridge

You might also like